ดูหนึ่งข้อความ
  #65  
Old 08 มกราคม 2005, 05:23
nooonuii nooonuii ไม่อยู่ในระบบ
ผู้พิทักษ์กฎทั่วไป
 
วันที่สมัครสมาชิก: 25 พฤษภาคม 2001
ข้อความ: 6,408
nooonuii is on a distinguished road
Post

ข้อความเดิมของคุณ aaaa:
--------------------------------------------------------------------------------------------
19. ให้ \( a_1,a_2,\ldots,a_n \) เป็นจุดบนวงกลมหนึ่งหน่วย \( D=\{z:|z|=1\} \) จงพิสูจน์ว่ามีจุด \( z_0\in D \) ซึ่ง
\[
|z_0-a_1|+|z_0-a_2|+\cdots+|z_0-a_n|\geq n
\]
--------------------------------------------------------------------------------------------

จะพิสูจน์โดยใช้ Lemma ต่อไปนี้

\( Lemma \) : ให้ a เป็นจำนวนเชิงซ้อน จะได้ว่ามีจำนวนเชิงซ้อน \( z_{0} \) ซึ่ง \( |z_{0}|=1 \) และ \( |z_{0}+a|\geq 1 \)

\( Proof \) : ให้ \( a = re^{i\theta} \) เลือก \( z_{0} = e^{i\theta} \) จะได้ z0 สอดคล้องเงื่อนไขตามต้องการ

ต่อไปจะพิสูจน์โจทย์ข้อ 19

ให้ \[ a = - (\frac{a_{1}+...+a_{n}}{n}) \]
โดย Lemma ข้างบน จะมี \( z_{0} \) บนวงกลมหนึ่งหน่วยซึ่งทำให้
\[ |z_{0} - \frac{a_{1}+...+a_{n}}{n}| \geq 1 \]
ดั้งนั้น \( |z_{0}-a_{1}|+...+|z_{0}-a_{n}| \geq |nz_{0} - (a_{1}+...+a_{n})| \geq n \)

P.S. สังเกตว่า จุด ai เป็นจุดใดๆบนระนาบเชิงซ้อนก็ได้
__________________
site:mathcenter.net คำค้น

08 มกราคม 2005 05:44 : ข้อความนี้ถูกแก้ไขแล้ว 4 ครั้ง, ครั้งล่าสุดโดยคุณ nooonuii
ตอบพร้อมอ้างอิงข้อความนี้